Modulesover PID
Modulesover PID
KEITH CONRAD
Every vector space over a field K that has a finite spanning set has a finite basis: it is
isomorphic to K n for some n ≥ 0. When we replace the scalar field K with a commutative
ring A, it is no longer true that every A-module with a finite generating set has a basis: not
all modules have bases. But when A is a PID, we get something nearly as good as that:
(1) Every submodule of An has a basis of size at most n.
(2) Every finitely generated torsion-free A-module M has a finite basis: M ∼ = An for a
unique n ≥ 0.
(3) Every finitely generated A-module M is isomorphic to Ad ⊕ T , where d ≥ 0 and T
is a finitely generated torsion module.
We will prove this based on how a submodule of a finite free module over a PID sits inside
the free module. Then we’ll learn how to count with ideals in place of positive integers.
1. Preliminary results
We start with two lemmas that have nothing to do with PIDs.
Lemma 1.1. If A is a nonzero commutative ring and Am ∼
= An as A-modules then m = n.
Proof. The simplest proof uses a maximal ideal m in A. Setting M = Am and N = An , if
M∼ = N as A-modules then it restricts to an isomorphism mM ∼ = mN and we get an induced
∼ m ∼ n
isomorphism M/mM = N/mN . This says (A/m) = (A/m) as A-modules, hence also as
A/m-vector spaces, so m = n from the well-definedness of dimension for vector spaces.
Lemma 1.1 says all bases in a finite free module over a nonzero commutative ring have the
same size, and we call the size of that basis the rank of the free module. So the term rank
means dimension when the ring is a field.1 The proof of Lemma 1.1 is valid for free modules
with an infinite basis, so the rank of a free module with an infinite basis is well-defined as
a cardinal number. For our purposes, we only care about free modules of finite rank.
Commutativity in Lemma 1.1 matters: for some noncommutative A, A2 ∼ = A as left
A-modules.
Lemma 1.2. A finitely generated torsion-free module M over an integral domain A embeds
into a finite free A-module. More precisely, if M 6= 0, there is an embedding M ,→ Ad for
some d ≥ 1 such that the image of M intersects each standard coordinate axis of Ad .
Proof. Let K be the fraction field of A and x1 , . . . , xn be a generating set for M as an
A-module. We will show n is an upper bound on the size of each A-linearly independent
subset of M . Let f : An → M be the linear map where f (ei ) = xi for all i. (By e1 , . . . , en
we mean the standard basis of An .) LetPy1 , . . . , yk be linearly independent in M , so their
A-span is isomorphic to A . Write yj = ni=1 aij xi with aij ∈ A. We pull the yj ’s back to
k
1The word “rank” means something completely different in linear algebra – the dimension of the image
of a linear map.
1
2 KEITH CONRAD
Letting x run through the spanning set x1 , . . . , xn there is an a ∈ A − {0} such that
axi ∈ dj=1 Atj for all i, so aM ⊂ dj=1 Atj . Multiplying by a is an isomorphism of M
P P
with aM , so we have the sequence of A-linear maps
d
X
M → aM ,→ Atj → Ad ,
j=1
containment ki=0 Axi ⊂ M is clear. For the reverse containment, pick an arbitrary x ∈ M
P
and the previous paragraph tells us π(x) = a1 π(x1 ) + · · · + ak π(xk ) = π(a1 x1 + · · · + ak xk )
for some a1 , . . . , ak in A. Therefore x − ki=1 ai xi ∈ ker(π|M ), so x − ki=1 ai xi = a0 x0 for
P P
The case n = 0 is trivial and the case n = 1 follows from all submodules of A being (0)
or principal with a nonzero generator, and Aa ∼ = A as A-modules when a is nonzero in A
since A is an integral domain.
Suppose n > 1 and the theorem is proved for all submodules of An . For a submodule M of
An+1 , to show M is free of rank at most n + 1 write An+1 = An ⊕ A and let π : An ⊕ A An
be projection to the first component. Since π(M ) is a submodule of An , π(M ) is free of
rank 6 n by the inductive hypothesis.
Case 1: π(M ) = 0. Here M ⊂ 0 ⊕ A, so M is free of rank at most 1 since A is a PID.
Case 2: π(M ) 6= 0. Here π(M ) is a nonzero submodule of An , so π(M ) is free of positive
rank d ≤ n. Write a basis of π(M ) as π(e1 ), . . . , π(ed ) where ei ∈ M : π(M ) = di=1 Aπ(ei ).
L
The
P elements e1 , . . . , ed in M are linearly
P independent since their images under π are: if
ai ei = 0 in M then apply π to get ai π(ei ) = 0 in π(M ), so all ai are 0.
For m ∈ M , π(m) = di=1 ai π(ei ) for unique a1 , . . . , ad ∈ A. Then π(m − di=1 ai ei ) = 0,
P P
and both maps are linear (check!). So M ∼ = Ad ⊕ ker(π|M ) as A-modules. Thus M is free
of rank d or d + 1 (depending on ker(π|M ) being {0} or not), and d + 1 ≤ n + 1.
Theorem 2.2 is always false if A is not a PID, even for the A-module A itself.
Non-example 2.3. If A is not a PID then either it is not an integral domain or it has a
nonprincipal ideal. If A is not an integral domain then we have xy = 0 for some nonzero
x and y in A, and the principal ideal Ax is not a free A-module. If A has a nonprincipal
ideal then that ideal is not a free A-module.
Remark 2.4. Theorem 2.2 holds for infinite-rank free modules too: submodules of all free
modules over a PID are free. The proof for infinite bases uses Zorn’s lemma [5, pp. 650–651].
Remark 2.5. Unlike vector spaces, a spanning set of a finite free A-module might not
contain a basis: Z is spanned by {2, 3} as a Z-module, but {2} and {3} are not Z-bases.2
An analogue of that in Z2 is in Example 5.25.
Corollary 2.6. When A is a PID, every finitely generated torsion-free A-module is a finite
free A-module.
Proof. By Lemma 1.2, such a module embeds into a finite free A-module, so it is finite free
too by Theorem 2.2.
The term “free” in “torsion-free module” and “free module” means different things: a
torsion-free module has no nonzero torsion elements (all elements have annihilator ideal (0)
aside from the element 0), while a free module has a basis. So Corollary 2.6 is saying a
finitely generated module over a PID that has no torsion elements admits a basis. Corollary
2.6 is false without the finite generatedness hypothesis. For example, Q is a torsion-free
abelian group but it has no basis over Z: every (nonzero) free Z-module has proper Z-
submodules (that is, proper subgroups) of finite index while Q does not.
2 If you know about DVRs (a special type of PID): every minimal spanning set of a finite free module
over a DVR is a basis. The proof uses Nakayama’s lemma.
4 KEITH CONRAD
There is a convenient way of picturing a submodule of a finite free module over a PID:
bases can be chosen for the module and submodule that are aligned nicely, as follows.
Pictures will show how aligned bases look before seeing the main theorem about them.
The obvious Z-bases for M and M 0 are {1, i} and {1 + 2i, −2 + i}. In Figure 1, we shade
a box having each basis as a pair of edges and translate each box across the plane. The
modules M and M 0 are the intersection points of the networks of lines formed by the small
and large boxes, respectively. The lines are only a visual aid, showing how a choice of basis
gives a specific way to generate the module by the basis.
To see a completely different picture of the same two modules M and M 0 , we use new
bases: {1 + 2i, i} for M and {1 + 2i, 5i} for M 0 . These are bases because of the relations
1 + 2i 1 2 1 1 + 2i 1 0 1 + 2i
= , = ,
i 0 1 i 5i 2 1 −2 + i
where the two matrices are integral with determinant 1, so the elements of Z[i] in the
vector components on both sides have the same Z-span. These new bases lead to Figure 2,
where the parallelograms with each basis as a pair of edges is shaded and looks quite unlike
the shaded boxes of Figure 1. Translating the parallelograms across the plane produces
two new networks of lines (both sharing all the vertical lines) The intersection points are
the same as before; make sure you can see the vertices of the large box from Figure 1 as
intersection points of lines in Figure 2. In Figure 2 five of the parallelograms for M fill up
of the parallelograms for M 0 . These bases are aligned.
Theorem 2.14. Each finite free A-module M of rank n > 1 and nonzero submodule M 0
of rank m 6 n admit a pair of aligned bases: there is a basis v1 , . . . , vn of M and nonzero
a1 , . . . , am ∈ A such that
n
M m
M
0
M= Avi and M = Aaj vj .
i=1 j=1
Since A is a PID, Aa1 + · · · + Aam = (a) for some nonzero a ∈ A, so ϕ(M 0 ) ⊂ (a) for all ϕ.3
Write a = c1 a1 + · · · + cm am with cj ∈ A (the
P choice∨ of cj ’s may not be unique, but
fix such a representation for a) and define ψ = m j=1 cj ej as a linear map M → A. Then
Pm Pm Pm
ψ( j=1 aj ej ) = j=1 cj aj = a and j=1 aj ej ∈ M 0 , so
(a) ⊂ ψ(M 0 ) ⊂ (a).
Thus ψ(M 0 ) = (a) and this ideal is the unique maximal member with respect to inclusion
among all ideals ϕ(M 0 ) as ϕ varies over all linear maps M → A. (This is not saying (a) is
a maximal ideal!). This ends the motivation.
Step 1. The set of ideals S := {ϕ(M 0 ) : ϕ : M → A is linear} is not {(0)} and has a
maximal member (a).
Let {v1 , . . . , vn } be a basis of M . Since M 0 6= {0}, at least one coordinate function vj∨
for this basis of M is not identically 0 on M 0 , which makes vj∨ (M 0 ) a nonzero ideal in S.
Each nonzero ideal of A is contained in only finitely many ideals of A since A is a PID: if
(x) is a nonzero ideal then we have (x) ⊂ (y) if and only if y | x. Up to unit multiples, there
are only finitely many possible y since x has only finitely many factors up to unit multiples.
Applying this to (x) = ϕ(M 0 ) for some ϕ where ϕ(M 0 ) 6= (0) and looking at the finitely
many ideals containing (x), S contains a maximal member with respect to inclusion, say
(a) = ψ(M 0 ).
Then a 6= 0, and maximality means that if (a) ⊂ ϕ(M 0 ) for some ϕ, then ϕ(M 0 ) = (a).4
Since a ∈ ψ(M 0 ), there’s some v 0 ∈ M 0 such that a = ψ(v 0 ) .
Step 2: For the ideal (a) in Step 1 and each A-linear map ϕ : M → A, a | ϕ(v 0 ) in A.
Write the ideal (a, ϕ(v 0 )) as (b), where b ∈ A − {0}. For some x and y in A,
b = ax + ϕ(v 0 )y = xψ(v 0 ) + yϕ(v 0 ) = (xψ + yϕ)(v 0 ).
3If a | a | · · · | a , which we don’t want to assume, then we can let a = a since (a ) ⊂ · · · ⊂ (a ) ⊂ (a ).
1 2 m 1 m 2 1
4We anticipate (a) will be the unique maximal member of S by the argument in the motivational section
at the start of this proof, but at the moment (a) is just some maximal member of S.
8 KEITH CONRAD
Proof. For each g ∈ G, g(An ) is a finite free A-submodule of K n . Let M be the A-module
n n n
P
g∈G g(A ), which is the set of finite sums of vectors in K that belong to g(A ) for some
g ∈ G. This is an A-module in K n that contains An (use g = In ). Note M is G-stable
(carried back to itself when acting on it by elements of G).
10 KEITH CONRAD
The common denominator hypothesis says there is a nonzeroPa ∈ A such that each
matrix ag for g ∈ G has all of its entries in A. Therefore aM = g∈G (ag)(An ) ⊂ An , so
M ⊂ (1/a)An in K n . Thus An ⊂ M ⊂ (1/a)An . By Corollary 2.7, M ∼ = An as A-modules.
Let ϕ : An → M be an A-module isomorphism. Then M = ϕ(An ) is the A-linear span
of ϕ(e1 ), . . . , ϕ(en ). These vectors are A-linearly independent since they span a free A-
module of rank n, so they are also K-linearly independent: a nontrivial K-linear relation
would become a nontrivial A-linear relation. Therefore the matrix Φ = [ϕ(e1 ) · · · ϕ(en )] is
invertible, so Φ ∈ GLn (K) and Φ = ϕ on An .
For each g ∈ G, gM ⊂ M since M is G-stable, so gϕ(An ) ∈ ϕ(An ). Therefore g(Φ(An )) ⊂
Φ(An ), so Φ−1 gΦ(An ) ⊂ An . This means the matrix Φ−1 gΦ has entries in A (its columns
are (Φ−1 gΦ)(ei ) for i = 1, . . . , n). Letting g run over G, the group Φ−1 GΦ is conjugate to
G and its elements have matrix entries in A.
Our second application to matrix groups will be the computation of normalizers. For a
group G and subgroup H of G, the normalizer of H is NG (H) = {g ∈ G : gHg −1 = H} =
{g ∈ G : g −1 Hg = H}: this is the largest subgroup of G in which H is a normal subgroup.
Theorem 3.6. Let A be a PID and K be its fraction field. In the group GLn (K), both
GLn (A) and SLn (A) have normalizer K × GLn (A).
We view K × inside GLn (K) as the nonzero scalar diagonal matrices {cIn : c ∈ K × }.
This is the center of GLn (K).
Proof. The result is trivial when n = 1, since GL1 (K) = K × is commutative, so from now
on we can assume n ≥ 2. Set G = GLn (K). We will show NG (GLn (A)) = K × GLn (A) and
then indicate how that argument can be modified to show NG (SLn (A)) = K × GLn (A).
That K × GLn (A) ⊂ NG (GLn (A)) is obvious. To prove NG (GLn (A)) ⊂ K × GLn (A), pick
g ∈ NG (GLn (A)). We will find c ∈ K × such that cg ∈ GLn (A). The proof is based on an
answer to the Mathoverflow question https://mathoverflow.net/questions/80667. Set
M = g(An ).
Step 1: M is a free A-module of rank n and is GLn (A)-stable: L(M ) ⊂ M for all
L ∈ GLn (A).
The A-module M is spanned by g(e1 ), . . . , g(en ) where e1 , . . . , en is the standard basis
of An , and g(e1 ), . . . , g(en ) is A-linearly independent since g is an invertible matrix over A.
Thus M ∼ = An as A-modules.
To show M is GLn (A)-stable, we use the property g −1 GLn (A)g = GLn (A), which is the
condition of g normalizing GLn (A). For L ∈ GLn (A), L0 := g −1 Lg is in GLn (A) too, so
Lg = gL0 . Therefore
L(M ) = Lg(An ) = gL0 (An ) = g(An ) = M.
Step 2: There is d ∈ A − {0} such that dM ⊂ An .
Let d ∈ A − {0} be a common denominator for the coordinates of g(e1 ), . . . , g(en ) in K n .
Then dg(ei ) ∈ An for i = 1, . . . , n, so dM ⊂ An .
Step 3: There is c ∈ K × such that cg ∈ GLn (A).
By Steps 1 and 2, dM is a submodule of An with rank n, so by Theorem 2.14 we can find
a basis {f1 , . . . , fn } of An and nonzero a1 , . . . , an in A such that {a1 f1 , . . . , an fn } is a basis
of dM and a1 | a2 | · · · | an . We won’t use the full strength of that divisibility condition,
but just a1 | ai for i = 1, . . . , n.
MODULES OVER A PID 11
Proof. A finitely generated module in Theorem 4.1 is a torsion module if and only if F = 0.
The description of T in Theorem 4.1 gives the desired cyclic decomposition for finitely
generated torsion modules.
Corollary 2.6 could be regarded as a consequence of Theorem 4.1: a finitely generated
module is F ⊕ T where F is finite free and T is torsion, and being torsion-free forces T = 0,
so the module is free.
When a finitely generated A-module is written as F ⊕T , where F is a finite free submodule
and T is a torsion submodule, the choice of F is not unique but T is unique: T is the set
of all elements in F ⊕ T with nonzero annihilator ideal, which is a description that makes
no reference to the direct sum decomposition. The best way to see F is not unique is by
examples.
Example 4.4. Using A = Z, Z×Z/(2) has generating set {(1, 0), (0, 1)} and {(1, 1), (0, 1)}.
Therefore it can be written as F1 ⊕ T1 , where F1 = h(1, 0)i ∼ = Z and T1 = 0 ⊕ Z/(2), and
∼
also as F2 ⊕ T2 where F2 = h(1, 1)i = Z and T2 = 0 ⊕ Z/(2) = T1 .
√ √
Example 4.5. It can be shown that every√unit in Z[ 2] has the form ±(1 + 2)k some
choice of sign ±1 and some ×
√ × integer k, so Z[ 2] √is a finitely generated√ abelian group. The
torsion subgroup of Z[ 2] is {±1}, while 1 + 2 and −(1 + 2) each generate different
free subgroups that complement {±1}:
√ √ √
Z[ 2]× = {±1} × (1 + 2)Z = {±1} × (−(1 + 2))Z
√ ×
This leads to two isomorphisms√ of Z[ 2] with Z×Z/2Z, which √ identify different subgroups
with Z (the powers of 1 + 2 and the powers of −(1 + 2)) but both identify the same
subgroup {±1} with Z/(2).
√ √ × √ Z √ Z √ √ Z
Example 4.6. It can √ be shown
√ that
√ Z[ √2, 3] = ±(1 + 2) (2 +√ 3) ( 2 +
√ 3) ,
where
√ the
√ units 1 + 2, 2 + 3, and 2 + 3 (with respective
√ inverses 2
√ b √ − 1, 2 +√ 3, and
3 − 2) have no multiplicative relations over Z: if (1 + 2)a (2 √ + √ 3) ( 2 + 3)c = 1
for integer exponents a, b, and c, then a = b√ = c√= 0. Therefore√Z[ 2, √3]×√ ∼
= Z3 √ × Z/2Z.
Two √ examples of complements to ±1 in Z[ 2, 3]× are h1 + 2, 2 + 3, 2 + 3i and
√ √ √
h1 + 2, −(2 + 3), −( 2 + 3)i.
While the free part of a direct sum decomposition is not unique, the rank of the free
part is unique: writing M = F ⊕ T with F finite free and T necessarily being the torsion
submodule Mtor of M , we have F ∼ = M/T = M/Mtor so the rank of F is the rank of the
finite free module M/Mtor . The rank of a finitely generated module over a PID A is defined
to be the rank of its free part, which is well-defined even though the free part itself is not.
In down-to-earth terms, the rank is the largest number of linearly independent torsion-free
elements (over Z, it is the largest number of independent elements of infinite order).
Our focus in this section was on describing the abstract structure of a finitely generated
module over a PID, not proving a module over a PID arising somewhere in mathematics is
finitely generated. The fact that certain abelian groups (Z-modules) are finitely generated
can be a major theorem (look up the Mordell–Weil theorem) and a formula for the rank
can be a major theorem or conjecture (look up Dirichlet’s unit theorem or the Birch and
Swinnerton-Dyer conjecture).
Theorem 4.1 says for a finitely generated module M over a PID A, M = Mtor ⊕ F where
F is a free A-module. In particular, Mtor is a direct summand of M . That conclusion can
MODULES OVER A PID 13
become false if (i) A is a PID and M is not finitely generated or (ii) M is finitely generated
and A is not a PID. We’ll give examples of this using A = Z for (i) and A = Z[x] for (ii).
Q
Example 4.7. Set M = p Z/(p), which is a Z-module in a natural way. We will show
the torsion submodule T of M is not a direct summand: M 6= T ⊕ N for a submodule N .
The argument is Lbased on [4, Theorem 10.2].
Step 1: T = p Z/(p) and T 6= M .
To prove the claim, one containment is immediate: since elements in the direct sum have
only finitely many nonzero coordinates and each Z/(p) is killed off by a single prime p, each
element of the direct sum is killed off by a product of finitely many primes and thus is in
T . Conversely, if m := (ap mod p)p is an element of T and km = 0 where k ∈ Z − {0}, then
kap ≡ 0 mod p for each prime p. When p - k, ap ≡ 0 mod p, so m has only finitely many
possible
L nonzero coordinates (its p-coordinates where p is a prime factor of k). Therefore
m ∈ p Z/(p), which finishes Step 1. Because there are infinitely many primes, T 6= M .
Step 2: We can’t write M = T ⊕ N for a Z-submodule N of M .
Assume there is such a direct sum decomposition. Then N ∼
= M/T , so M has a submodule
isomorphic to M/T . We will get a contradiction by showing M and its submodules all share
a property that is not true
T for M/T .
The property is this: p pM = {0}, where the intersection runs over all primes. Indeed,
for each prime
T p the p-coordinate of an element of pM has to be 0, T so all coordinates of an
element of p pM equal 0. Therefore if N is a submodule of M , p pN = {0}.
T
In contrast to that, we will show p p(M/T ) 6= {0}. Specifically, let v := (1, 1, 1, . . .) be
the element of M with p-coordinate 1 mod p for each prime p. We’ll show
(i) v 6∈ T
T, so v 6= 0 in M/T ,
(ii) v ∈ p p(M/T ).
Proof of (i): If v ∈ T , then kv = 0 for some nonzero integer k since v 6= 0. Looking at
the coordinates of kv, we get k ≡ 0 mod p for each prime p, so k has infinitely many prime
factors and that forces k = 0, a contradiction.
Proof of (ii): Fix a prime p. We will find a w ∈ M and t ∈ T (depending on p). such
that v = pw + t, so v ≡ pw mod T , so v ∈ p(M/T ).
All the coordinates of v are 1. For each prime q other than p, p is invertible mod q so
we can define wq ∈ Z/(q) by the congruence pwq ≡ 1 mod q. Define wp = 0 mod p. Then
w = (wq )q is in M and pw has q-coordinate 1 for each q 6= p and pw has p-coordinate
0. Therefore v = pw + t where t has p-coordinate 1 and every other coordinate 0, which
means t ∈ T . That completes Step 2.
Remark 4.8. This construction of a Z-module whose torsion submodule is not a direct
summand of it carries over without change to modules over a PIDQA that has infinitely
many non-associate irreducible elements. Let M be the A-module (π) A/(π), where the
direct product is taken over all distinct
L maximal ideals (π). It is left to the reader to prove
the torsion submodule T of M is (π) A/(π) and M 6= T ⊕ N for an A-submodule N of M
by an argument similar to the Q case A = Z above.
The infinite direct product p Z/(p) in Example 4.7 may seem artificial. What about
using the abelian group S 1 , whose torsion submodule is the group µ∞ of all roots of unity?
It seems plausible that we can’t write S 1 = µ∞ × H for a subgroup H of S 1 , but that is
possible by the Axiom of Choice since µ∞ is a divisible abelian group. See Corollary 2.5 in
14 KEITH CONRAD
https://kconrad.math.uconn.edu/blurbs/zorn1.pdf.
L Note that unlike µ∞ , the torsion
submodule p Z/(p) in Example 4.7 is not divisible.
Example 4.9. Let A = Z[x] and r be an integerbesides 0 and ±1, so the ideal a := (r, x)
in A is not principal. We will show M := A2 /a xr , which is a finitely generated A-module,
does not have Mtor as a direct summand. This example and the argument behind it are
from https://math.stackexchange.com/questions/3593455/.
Since A is an integral domain and a is a proper ideal in A, xr 6∈ a xr , so xr 6= 0 in M .
Claim: Mtor = A xr .
To prove the claim, we have xr ∈ Mtor since x xr = 0 and x 6= 0. (Also r xr = 0.) Thus
A xr ⊂ Mtor . For the reverse containment, let ab ∈ Mtor for a and b in A, so c ab = 0 for
If d = 0 then ca and cb are 0, so a and b are 0 since c is nonzero and Z[x] is an integral
domain. Thus ab = 0 ∈ A xr .
!
r 1 0 1 0 r r
f =f r +x = rf + xf = ra + xb =0
x 0 1 0 1 x x
r
since x is killed by both r and x.
Remark 4.10. Example 4.9 works in the same way if we replace Z[x] by R[x] where R is
an integral domain that is not a field and r is a nonzero nonunit in R. What we used about
r and x in the example is that (r, x) is a proper ideal of Z[x], x is prime in Z[x], and x - r.
It wasn’t important that r is an integer. (For instance, we could have used r = x + 2).
Therefore we could replace Z[x] by an integral domain A containing a proper ideal of the
form (r, p) where p is prime
√ in A and p - r. Such a domain A consisting of numbers rather
than √
polynomials√is Z[ n] where the integer n is not a square and √ n ≡ 1 mod 4: √ the ideal
(1 + n, 2) in Z[ n] is proper (it has index 2), 2 is prime in Z[ n], and 2 - (1 + n).
There are some integral domains A that are not a PID but every finitely generated A-
module M has Mtor as a direct summand. This is true if A is a Dedekind domain, which is a
type of generalization of a PID. When we write M = Mtor ⊕ N , the complementary module
N is torsion-free but need not be free (that is, it need not have an A-basis). For example,
√ √ 1+√−3 √ √
let A = Z[ −6] and M = Z[ 2, 2 ], so A ⊂ M since M contains 2 and −3. Then
Mtor = {0} and M is finitely generated as an A-module (it is already finitely generated as
MODULES OVER A PID 15
a Z-module, and Z ⊂ A), but it can be shown that M is not a free A-module. See Example
2.3 in https://kconrad.math.uconn.edu/blurbs/gradnumthy/notfree.pdf.
When A = Z and G is a finite abelian group, AnnZ (G) is generated by the least positive
integer whose power (or multiple, in additive notation) kills everything in the group and is
traditionally called the exponent of G. The size and exponent of G are equal exactly when
G is cyclic, and likewise cardA (T ) = AnnA (T ) exactly when T is a cyclic A-module.
Now we show cardA (T ) is well-defined. You may want to skip the proof on a first reading.
Theorem 5.7. If A/(a1 ) ⊕ · · · ⊕ A/(am ) ∼ = A/(b1 ) ⊕ · · · ⊕ A/(bn ) as A-modules, where the
ai ’s and bj ’s are nonzero, then (a1 a2 · · · am ) = (b1 b2 · · · bn ) as ideals.
Proof. If A is a field then the theorem is obvious (both ideals are (1)), so assume A is
not a field: it has irreducible elements. For each irreducible π in A, we will show the
highest powers of π in a1 a2 . . . am and b1 b2 · · · bn are equal. Then by unique factorization
(a PID is a UFD) a1 a2 . . . am and b1 b2 · · · bn are equal up to multiplication by a unit, so
(a1 a2 . . . am ) = (b1 b2 · · · bn ).
Set T = A/(a1 ) ⊕ · · · ⊕ A/(am ). For each irreducible π in A we look at the descending
chain of modules
T ⊃ πT ⊃ π 2 T ⊃ · · · ⊃ π i T ⊃ · · ·
The quotient of successive modules π i−1 T /π i T is an A-module on which multiplication by
π is 0, so this is an A/(π)-vector space. Since T is finitely generated, so is π i−1 T (multiply
the generators of T by π i−1 ) and thus so is its quotient π i−1 T /π i T , so π i−1 T /π i T is a finite-
dimensional A/(π)-vector space. The dimensions dimA/(π) (π i−1 T /π i T ) will be the key. We
will show the highest power of π in a1 a2 . . . am is i≥1 dimA/(π) (π i−1 T /π i T ).
P
Proof. Our proof will be an application of aligned bases in a finite free abelian group and
finite-index subgroup: Theorem 2.14 with A = Z.
The n-tuples x1 , . . . , xn and y1 , . . . , yn do not have the same Z-span (unless M 0 = M ),
but morally they should have the same Q-span. To make this idea precise, we transfer
the data of M and 0 n
n
PnM into the vector space Q . Let e1 , . . . , en be the standard basis of
Q and set fj = i=1 cij ei , using the same coefficients that express y1 , . . . , yn in terms of
x1 , . . . , xn . Identify M with Zn by xi ↔ ei (extended by Z-linearity). This isomorphism
identifies yj with fj for all j, so M 0 inside M is identified with the Z-span of f1 , . . . , fn
inside Zn .
A Z-linear map Zn → Zn is determined by its effect on the standard basis e1 , . . . , en of
n n n
Z . Let ϕ P:nZ → Z be the Z-linear map determined by ϕ(e1 ) = f1 , . . . , ϕ(en ) = fn . Then
ϕ(ej ) = i=1 cij ei for j = 1, . . . , n, so (cij ) is the matrix representation of ϕ with respect
to the standard basis e1 , . . . , en of Zn and
ϕ(Zn ) = Zϕ(e1 ) ⊕ · · · ⊕ Zϕ(en ) = Zf1 ⊕ · · · ⊕ Zfn ,
so [M : M 0 ] = [Zn : ϕ(Zn )]. We will show [Zn : ϕ(Zn )] = | det(ϕ)| = | det(cij )|.
The bases e1 , . . . , en and f1 , . . . , fn of Zn are usually not aligned with each other. By
Theorem 2.14, there is a set of aligned bases for Zn and its submodule ϕ(Zn ):
Zn = Zv1 ⊕ · · · ⊕ Zvn , ϕ(Zn ) = Za1 v1 ⊕ · · · ⊕ Zan vn ,
where the ai ’s are nonzero integers. Then
n
Zn /ϕ(Zn ) ∼
M
= Z/ai Z,
i=1
which tells us
(5.3) [Zn : ϕ(Zn )] = |a1 a2 · · · an | .
We want to prove | det(ϕ)| = |a1 a2 · · · an | too.
A Z-linearly independent set of size n in Qn is a Q-basis of Qn , so all four sets {ei },
{ϕ(ei )}, {vi } and {ai vi } are Q-bases for Qn . For two Q-bases of Qn there is a unique
Q-linear map Qn → Qn taking one basis to the other. The Q-linear map Qn → Qn taking
ei to ϕ(ei ) is the natural extension of ϕ : Zn → Zn from a Z-linear map to a Q-linear map,
so we will also call it ϕ (its matrix representation with respect to the standard basis of Qn
is (cij ), just like ϕ as a Z-linear map on Zn ). Consider the diagram of Q-linear maps
e_i
ϕ ϕ
Qn / Qn / fi
O O
α γ where α γ
_
Qn / Qn vi / ai vi
β β
6We are using Q-linear maps throughout because it is nonsense to talk about the determinant of a Z-
linear map Zn → ϕ(Zn ) when ϕ(Zn ) 6= Zn : the different bases don’t all have the same Z-span but they do
all have the same Q-span.
20 KEITH CONRAD
Using the Q-basis {vi } of Qn , the matrix representation [β] is diagonal with ai ’s along its
main diagonal, so
det(β : Qn → Qn ) = a1 a2 · · · an .
What is det(α)? The map α identifies two Z-bases of the same Z-module Zn :
α(c1 e1 + · · · + cn en ) = c1 v1 + · · · + cn vn , ci ∈ Z.
Therefore α is invertible as a Z-linear map of Zn to itself. Using {ei } as the basis in which
a matrix for α is computed, the matrices of α : Qn → Qn over Q and α : Zn → Zn over Z
are the same. Since an invertible Z-linear map Zn → Zn has determinant ±1,
det(α : Qn → Qn ) = det(α : Zn → Zn ) = ±1.
A similar argument shows
det(γ : Qn → Qn ) = det(γ : ϕ(Zn ) → ϕ(Zn )) = ±1
since γ identifies two Z-bases of ϕ(Zn ). Feeding these determinant formulas into the right
side of (5.4),
(5.5) det(ϕ) = ±a1 a2 · · · an .
Comparing (5.3) and (5.5), |det(ϕ)| = |a1 a2 · · · an | = [Zn : ϕ(Zn )] = [M : M 0 ].
Example 5.20. Let M = Z[i] = Z + Zi and M 0 = (1 + 2i)Z[i] = Z(1 + 2i) + Z(−2 + i). In
Example 2.13, we saw [M : M 0 ] = 5 by finding aligned Z-bases for M and M 0 . We will now
compute this index in a new way by using a determinant for a matrix expressing a Z-basis
of M 0 in terms of a Z-basis of M that are not aligned bases.
A Z-basis for Z[i] is {1, i} and a Z-basis for (1 + 2i) is {1 + 2i,
−2 + i}. The isomorphism
of abelian groups (Z-modules) Z[i] → Z2 where x + yi 7→ xy identifies the ideal (1 + 2i)
with Z 12 + Z −2 1 −2 2 5.19 [M : M 0 ] = det( 12 −2
1 = ( 2 1 )(Z ), so by Theorem
√ 1 ) = |5| = 5.
Similarly,
√ for a pure quadratic√ ring Z[ m] where m is an integer that √ is not a perfect
square, Z[ m] has Z-basis {1, √m} and a √ nonzero principal ideal√(a + b m) √ (for a, b ∈ Z
not both 0) has Z-basis {a + b m, mb + a m}, so the index [Z[ m] : (a + b m)] equals
a b ) = |a2 − mb2 |.
det( mb a
√ √ √
Example 5.21. In √ the ring Z[ 10] let a be the ideal (2+5 10, 4+7 10). We √ will compute
the index of a in Z[√ 10] as abelian
√ groups using (unaligned) Z-bases
√ for Z[ √ 10] and a.
A Z-basis for Z[ 10] is {1, 10}. A Z-basis for a is {2 + 5 10, 4 + 7 10}, but this
requires verification because it is a stronger condition to generate a as a Z-module than to
generate it as an ideal7: the initial definition of a tells us that
√ √ √ √
a = Z[ 10](2 + 5 10) + Z[ 10](4 + 7 10)
√ √ √ √
= (Z + Z 10)(2 + 5 10) + (Z + Z 10)(4 + 7 10)
√ √ √ √
= Z(2 + 5 10) + Z(50 + 2 10) + Z(4 + 7 10) + Z(70 + 4 10).
√ √
For a to be spanned over Z by 2 + 5 10 and 4 + 7 10, we need to write the other two
Z-module generators in terms of them. After some linear algebra, we can do this:
√ √ √
50 + 2 10 = −57(2 + 5 10) + 41(4 + 7 10),
√ √ √
70 + 4 10 = −79(2 + 5 10) + 57(4 + 7 10).
7For example, in Z[i], (17, 3 + 5i) 6= Z17 + Z(3 + 5i) since the ideal contains 4 + i = 17i − (3 + 5i)(2 + 2i)
but 4 + i 6= 17a + (3 + 5i)b for integers a and b: look at the imaginary parts.
MODULES OVER A PID 21
√ √ √ √
Therefore a = Z(2 + 5 10) + Z(4 + 7 10) and the numbers 2 + 5 10 and 4 + √ 10 are2
7
obviously Z-linearly
√ independent, so they are a Z-basis of a. The isomorphism Z[ 10] → Z
where x + y 10 7→ xy identifies the ideal a with Z 25 + Z 47 = ( 25 47 )(Z2 ), so Theorem
√
5.19 tells us that the index of a in Z[ 10] is |det( 25 47 )| = | − 6| = 6. The absolute value is
important: the index is not −6.
Theorem 5.22. Let M be a finite free module over the PID A with rank n and M 0 be a
submodule with
Pnrank n. Let x1 , . . . , xn be a basis of M and y1 , . . . , yn be a basis of M 0 .
0
Writing yj = i=1 cij xi with cij ∈ A, (det(cij )) = [M : M ]A . In particular, det(cij ) 6= 0.
Proof. Let K be the fraction field of A. If we run through the proof of Theorem 5.19 with
A in place of Z, K in place of Q, and use alignedLbases v1 , . . . , vn and a1 v1 , . . . , an vn for
M and M 0 , with all ai nonzero in A, so M/M 0 ∼ n
= i=1 A/(ai ), then the proof of Theorem
5.19 shows det(cij ) = det ϕ = ua1 . . . an , where u = det α det γ is a unit in A. The K-linear
operators α and γ on K n have unit determinant since they are also A-linear operators on
An and ϕ(An ) sending a basis to a basis.
By the definition of A-index, [M : M 0 ]A = cardA (M/M 0 ) = (a1 a2 · · · an ).
Corollary 5.23. Let M bePa finite free A-module with rank n and basis x1 , . . . , xn . For
y1 , . . . , yn in M , write yj = ni=1 cij xi with cij ∈ A. Then y1 , . . . , yn is linearly independent
if and only if det(cij ) 6= 0.
Proof. If y1 , . . . , yn is a linearly independent set, then its A-span in M is a free A-submodule
of rank n, so det(cij ) 6= 0 by Theorem 5.22.
Conversely, if det(cij ) 6= 0, then we want to show an A-linear relation nj=1 cj yj = 0 with
P
cj ∈ A must have all cj equal to 0. Writing yj in terms of the xi ’s,
n n n n n
!
X X X X X
0= cj yj = cj cij xi = cij ci xi ,
j=1 j=1 i=1 i=1 j=1
Pn
so looking at the coefficients of x1 , . . . , xn tells us j=1 cij ci = 0 for all i. As a matrix
equation this says
c11 c21 . . . cn1 c1 0
c12 c22 . . . cn2 c2 0
.. .. = .. .
.. .. . .
. . . . . .
c1n c2n . . . cnn cn 0
n n
This is an equation in A ⊂ K , where K is the fraction field of A. The matrix is the
transpose of (cij ). Since det((cji )> ) = det(cij ) 6= 0 the vector vanishes, so all ci are 0.
Corollary 5.24. If M is a finite free module over the PID A with basis x1 , . . . , xn , a set
of n elements y1 , . . . , yn in M is a basis of M if and only if the matrix (cij ) expressing the
y’s in terms of the x’s has unit determinant.
Proof. If y1 , . . . , yn is a basis of M then (det(cij )) = [M : M ]A = (1) by Theorem 5.22,
so det(cij ) ∈ A× . Conversely, if det(c ×
P ij ) ∈ A then y1 , . . . , yn is linearly
P independent by
Corollary 5.23 and the A-index of Ayj in M is (det(cij )) = (1), so Ayj = M .
Example 5.25. The Z-span of 22 , 36 , and 05 is Z2 since 10 = −4 22 + 3 36 − 2 05
det ( 22 05 ) = 10, and det ( 36 05 ) = 15. This is a rank-2 analogue of Remark 2.5.
22 KEITH CONRAD
where each quotient module π i−1 T /π i T is a vector space over the field A/(π), and the sum
is finite since the ith term is 0 for large enough i (depending on T ). This suggests the
following weaker notion of size for finitely generated torsion modules.
Definition 5.27. For a finitely generated torsion module T over the PID A, set the primary
cardinality of T to be
X
ωA (T ) = dimA/(π) (T /πT ) ∈ {0, 1, 2, 3, . . .}
(π)
where the sum runs over all nonzero prime ideals (π) of A.
Example 5.28. Let A = Z and T = Z/100Z ∼ = Z/4Z ⊕ Z/25Z. If p is 2 or 5 then
T /pT = (Z/100Z)/(pZ/100Z) ∼ = Z/pZ, so dimZ/(p) (T /pT ) = 1. If p is a prime other than
2 or 5 then pT = T (p is invertible modulo 100), so T /pT = 0 and dimZ/(p) (T /pT ) = 0.
Thus
X
ωZ (Z/100Z) = dimZ/(p) (T /pT ) = dimZ/(2) (T /2T ) + dimZ/(5) (T /5T ) = 1 + 1 = 2.
prime p
For general n, ωZ (Z/nZ) is the number of prime factors of n, which explains the name
“primary cardinality” for ωA (T ). (In number theory, ω(n) is the standard notation for the
number of prime factors of n.) The multiplicity of the prime factors of n does not play
a role in ωZ (Z/nZ). For example, if we run through the same calculation for Z/10Z and
10Z/100Z (both cyclic of order 10) then ωZ (Z/10Z) = 2 and ωZ (10Z/100Z) = 2. So unlike
the ordinary notion of size, a finite abelian group and a proper subgroup (like Z/100Z and
its subgroup 10Z/100Z) could have the same primary cardinality.
In the sum defining ωA (T ), each term dimA/(π) (T /πT ) is finite since T being finitely
generated as an A-module makes T /πT finitely generated as an A/(π)-vector space. The
following lemma explains why the sum defining ωA (T ) has only finitely many nonzero terms.
Theorem 5.29. Let T be a finitely generated torsion A-module. The ideal AnnA (T ) =
{a ∈ A : aT = 0} is not (0) and dimA/(π) (T /πT ) > 0 if and only if π | AnnA (T ), which
occurs for only finitely many prime ideals (π).
In the paragraph following Example 5.6, we saw AnnA (T ) 6= (0) by a formula for this
ideal in terms of a cyclic decomposition of T . The point of reproving AnnA (T ) 6= (0) again
is to see it can be done without relying on a cyclic decomposition.
MODULES OVER A PID 23
Proof. To show AnnA (T ) 6= (0) we will use a finite spanning set of T as an A-module, say
{t1 , . . . , tk }. Since each element of T is an A-linear combination of t1 , . . . , tk , for a ∈ A we
have aT = {0} if and only if at1 = 0, . . . , atk = 0. Since T is a torsion module, each of
t1 , . . . , tk is killed by some nonzero element of A, and the product of such elements is a single
nonzero element of A killing all of t1 , . . . , tk and thus killing all of T . Thus AnnA (T ) 6= (0).
The ideal AnnA (T ) is principal, so it has a generator that is not 0, say AnnA (T ) = (α).
We will show dimA/(π) (T /πT ) > 0 if and only if π | α, or equivalently dimA/(π) (T /πT ) = 0
if and only if π - α.
If π - α then (α, π) = (1) since π is prime, so αx + πy = 1 for some x and y in A. Thus
for each t ∈ T , t = (αx + πy)t = x(αt) + π(yt) = π(yt) since αt = 0. This shows T ⊂ πT ,
and the reverse containment is obvious, so πT = T and thus T /πT = 0.
If π | α, write α = πα0 . Then α0 6∈ (α) (since α - α0 ), so α0 T 6= {0}: for some t ∈ T we
have α0 t 6= 0. We can’t have t ∈ πT since then α0 t ∈ α0 πT = αT = {0}, which isn’t true.
So T /πT 6= {0}.
We have dimA/(π) (T /πT ) > 0 if and only if T /πT 6= 0, and we showed that is the same
as π | α. A nonzero element of A has only finitely many prime factors up to unit multiple,
so π | α for only finitely many prime ideals (π).
Corollary 5.30. We have ωA (T ) = 0 if and only if T = 0.
Proof. Obviously if T = 0 then ωA (T ) = 0. Conversely, if ωA (T ) = 0 then for all prime
elements π we have dimA/(π) (T /πT ) = 0, so T = πT . By Theorem 5.29, the ideal AnnA (T )
has no prime factors, so AnnA (T ) = (1). That 1 ∈ AnnA (T ) implies 1 · T = 0, so T = 0.
Theorem 5.31. For finitely generated torsion A-modules T1 and T2 , ωA (T1 ⊕T2 ) = ωA (T1 )+
ωA (T2 ).
Proof. For each prime π, π(T1 ⊕ T2 ) = πT1 ⊕ πT2 , so (T1 ⊕ T2 )/π(T1 ⊕ T2 ) ∼
= (T1 /πT1 ) ⊕
(T2 /πT2 ). Isomorphic vector spaces have the same dimension, so
dimA/(π) (T1 ⊕ T2 )/π(T1 ⊕ T2 ) = dimA/(π) ((T1 /πT1 ) ⊕ (T2 /πT2 ))
= dimA/(π) (T1 /πT1 ) + dimA/(π) (T2 /πT2 ),
and summing over all (π) gives us ωA (T1 ⊕ T2 ) = ωA (T1 ) + ωA (T2 ).
Example 5.32. For a ∈ A − {0}, A/(a) is isomorphic to a direct sum of cyclic modules of
the form A/(π k ) for prime π by the Chinese remainder theorem:
a = uπ k1 · · · π kr =⇒ A/(a) ∼
1 r = A/(π k1 ) ⊕ · · · ⊕ A/(π kr ),
1 r
where u ∈ A×and π1 , . . . , πr are nonassociate primes in A. So each nonzero finitely gener-
ated torsion A-module T is isomorphic to a direct sum of cyclic torsion A-modules:
T ∼
= A/(π e1 ) ⊕ · · · ⊕ A/(π en ),
1 n
where π1 , . . . , πn are prime and e1 , . . . , en are positive integers. (The primes could be the
same when T is not cyclic, e.g.,P(Z/15Z)× = h2 mod 15i ⊕ h−1 mod 15i ∼ = Z/4Z ⊕ Z/2Z.)
n ej ej
By Theorem 5.31, ωA (T ) = j=1 ωA (A/(πj )). We have ωA (A/(πj )) = 1, so ωA (T ) =
n. Thus we have a concrete interpretation for ωA (T ): it is the number of terms in a
decomposition of T as a direct sum of cyclic modules having a prime-power annihilator
ideal (AnnA (A/(π e )) = (π e ) when π is prime). For instance, if a finite abelian group G is
expressed as a direct sum of cyclic abelian groups of prime-power order then ωZ (G) is the
number of direct summands, e.g., ωZ ((Z/15Z)× )) = ωZ ((Z/4Z) ⊕ Z/2Z) = 2.
24 KEITH CONRAD
Exercises.
1. Let A be a nonzero commutative ring. Show that if every submodule of every finite
free A-module is a free A-module, then A is a PID.
2. Suppose A is a PID and π is irreducible in A. Inside A2 , set
1 0 x 2
M =A +A 2 = : y ≡ 0 mod π
0 π y
and
π 1 x 2
N =A +A = : y ≡ 0 mod π, πx ≡ y mod π .
0 π y
a) Find a basis {e1 , e2 } of A2 and a1 and a2 in A such that {a1 e1 , a2 e2 } is a basis
of N . (Such an aligned pair of bases obviously exists for A2 and M .)
b) Show there is no basis {e1 , e2 } of A2 and a1 , a2 , b1 , b2 in A such that {a1 e1 , a2 e2 }
is a basis of M and {b1 e1 , b2 e2 } is a basis of N . That is, the submodules M and N
of A2 do not admit bases simultaneously aligned with a single basis of A2 .
3. Let A be a PID, M be a finite free A-module, and N be a submodule. Consider the
following conditions on M and N :
(i) the quotient module M/N is torsion-free,
(ii) every A-basis of N extends to an A-basis of M ,
(iii) some A-basis of N extends to an A-basis of M .
Corollary 2.9 shows (i) implies (ii), and trivially (ii) implies (iii). Prove (iii) implies
(i), so the three conditions are equivalent.
4. If M is a finitely generated module over a PID A and M 0 is a submodule, is it
always possible to align their decompositions into free parts and torsion parts: can
we write M = F ⊕ T and M 0 = F 0 ⊕ T 0 such that F and F 0 are free, T and T 0
are torsion, and F 0 ⊂ F and T 0 ⊂ T ? If A is not a field, show the answer is no by
picking an irreducible π in A and using M = A ⊕ A/(π) and M 0 = A(π, 1). (Hint:
first show M 0 is free.)
5. Let A be a PID.
a) Prove an analogue of Cauchy’s theorem from group theory: for a finitely
generated torsion A-module T and irreducible π in A such that π divides cardA (T ),
meaning π divides a generator of the ideal cardA (T ), show there is some t ∈ T with
“order” π: the annihilator ideal AnnA (t) = {a ∈ A : at = 0} is πA.
b) Let T and T 0 be finitely generated torsion A-modules such that cardA (T ) =
cardA (T 0 ). If f : T → T 0 is an A-linear map, show f is one-to-one if and only if it is
onto. (When A = Z this is the familiar statement that a homomorphism between
finite abelian groups of equal size is one-to-one if and only if it is onto.)
c) Show a finitely generated A-module M is a torsion module if and only if there
is some a 6= 0 in A such that aM = 0. (This is false without a hypothesis of finite
generatedness, e.g., Q/Z is a torsion abelian group and for all nonzero integers a
we have a(Q/Z) = Q/Z.)
d) For a finitely generated A-module M and submodule N , show M/N is a torsion
module if and only if there is some a 6= 0 in A such that aM ⊂ N .
6. Let A be a PID and R be a commutative ring containing A such that R is a finite free
A-module. Prove there is an A-basis of R that contains 1. (Hint: Let e1 , . . . , en be
an A-basis of R and write 1 = ni=1 ci ei and d = gcd(c1 , . . . , cn ). Prove ϕ(R) ⊂ dA
P
MODULES OVER A PID 25
for all A-linear maps ϕ : R → A. Deduce that d = 1 and then that there’s an
A-linear map ϕ : R → A such that ϕ(1) = 1. Conclude from the method of proof of
Theorem 2.14 that R = A ⊕ N for an A-submodule N of R. )
7. Let V be a finite-dimensional vector space over a field K and f : V → V be an
K-linear operator. Write Vf for V as a K[X]-module where X acts on V as f :
Xv = f (v) for v ∈ V .
a) Show cardK[X] (Vf ) = (χf (X)), where χf is the characteristic polynomial of f .
b) If χf (X) decomposes into linear factors in K[X], so all the eigenvalues of f
are in K and we can find a matrix for f in Jordan canonical form, show ωK[X] (Vf )
is the number of Jordan blocks in that matrix.
8. For a pair of finitely generated A-modules M ⊃ M 0 , show M/M 0 is a torsion module
if and only if M and M 0 have the same rank (that means the free parts of M and
M 0 have equal rank). This generalizes Corollary 2.16 in the case of free modules.
9. Let T be a finitely generated torsion module over the PID A and T 0 be a submodule.
Show cardA (T ) = cardA (T 0 ) cardA (T /T 0 ). In particular, if cardA (T 0 ) = cardA (T )
then cardA (T /T 0 ) = (1), so T /T 0 = {0} (Example 5.3) and thus T 0 = T , so if T 0 is
strictly contained in T then cardA (T ) ( cardA (T 0 ).
10. Let T be a finitely generated torsion module over the PID A. Write a cyclic decom-
position of T as A/(a1 ) ⊕ · · · A/(am ) for nonzero ai in A.
a) Show AnnA (T ) = (lcm(a1 , . . . , am )).
b) For t and t0 in T , let AnnA (t) = (a) and AnnA (t0 ) = (a0 ). Show there is an
A-linear combination of t and t0 whose annihilator ideal is (lcm(a, a0 )). (Hint: when
A = Z, this becomes the fact from group theory that in a finite abelian group, if g
has order m and h has order n, then some element of hg, hi has order lcm(m, n).)
c) Using (a) and (b), show there is a t0 ∈ T such that AnnA (t0 ) = AnnA (T ).
(Hint: when A = Z, this is the statement that in a finite abelian group, the least
common multiple of the orders of its elements is the order of some element of the
group.)
11. Prove Corollaries 5.23 and 5.24 if A is an integral domain, not necessarily a PID.
References
[1] P. Aluffi, Algebra: Chapter 0, Amer. Math. Soc., Providence, 2009.
[2] H. Cohen and H. W. Lenstra, Jr., “Heuristics on class groups of number fields,” pp. 33–62 in Number
theory, Noordwijkerhout 1983, Springer Lecture Notes in Mathematics 1068, Springer-Verlag, Berlin,
1984.
[3] S. Roman, Advanced Linear Algebra, 2nd ed., Springer-Verlag, New York, 2005.
[4] J. Rotman, An Introduction to the Theory of Groups,4th ed., Springer-Verlag, NY, 1995.
[5] J. Rotman, Advanced Modern Algebra, Prentice-Hall, Upper Saddle River, NJ, 2002.
[6] P. Samuel, Algebraic Theory of Numbers, Dover, 2008.